neeeeDanke, (edited )

thanks for looking it up:).

I do think the upper bound on that page is wrong thought. Incedentally in the article itself only the lower bound is prooven, but in its sources this paper prooves what I did in my comment before as well:

for the upper bound it has max +log(n) . (Section 2, eq 4) This lets us construct an example (see reply to your other comment) to disproove the notion about beeing able to calculate the max for many integers.

  • All
  • Subscribed
  • Moderated
  • Favorites
  • random
  • uselessserver093
  • Food
  • aaaaaaacccccccce
  • [email protected]
  • test
  • CafeMeta
  • testmag
  • MUD
  • RhythmGameZone
  • RSS
  • dabs
  • Socialism
  • KbinCafe
  • TheResearchGuardian
  • oklahoma
  • feritale
  • SuperSentai
  • KamenRider
  • All magazines